PT12.S4.Q22 - a long-term health study

Sarah889Sarah889 Alum Member
edited January 2017 in Logical Reasoning 877 karma

Hey All,

So we have this health study that found that people who gained approximately 1 pound per year after the age of 35 tended, on the whole, to live longer than those who maintained the weight they had at 35. However, there have been other studies that indicated that weight gain tends to lower life expectancy.

Pretty standard resolve the paradox stimulus. I'm thinking: "Okay...well 'weight gain' is a pretty vague concept. 1 pound a year versus 10 pounds a year will certainly yield different results. Maybe the answer choice has something to do with that? Or maybe we find out that one of the tested groups were not representative-- meaning maybe they were poor eaters or the weight gain was a result of muscle gain because they were regular gym goers."

Well, none of the answer choices did any of those things (not completely anyway).

I eliminated the correct answer (C) because it says that smokers tend to be leaner than nonsmokers. It says nothing about weight gain.

If (C) read: Smokers, who tend to be less likely to gain weight than non smokers, tend to have shorter life spans than nonsmokers, then I could see what the test writers were getting at. (C) would be providing us with an example of a situation in which another factor (namely, smoking) can effect the outcome of the study. (C) would give us a potential reason that these two studies yielded different results because the group was not a representative group. I think (C) would still provide a very weak resolution because it is not specifically linking the scenario in (C) with our stimulus and we are just given a vague possibility that may or may not be relevant. However, I'm not a fan of the other ACs either, so I guess (C) is what we're left with. The "may" part of "may or may not be relevant" is apparently enough.

HOWEVER, my contention with C is what I mentioned earlier. During my timed section (and even BR...) I eliminated C because it said nothing about weight gain. So smokers are usually leaner? Okay...? Maybe there is some sort of propensity in leaner people to smoke? Maybe people who eat less are more inclined to pick up smoking habits? The point is we don't know...so how on earth can we assume that weight gain is playing a factor at all in the scenario in (C)?

Thanks in advance.

Comments

  • SamiSami Live Member Sage 7Sage Tutor
    10774 karma

    I want to help. But I am at work and I dont' have the question on hand ;'( .

    If nobody has answered by the evening, I'll take a look :smiley: <3

  • lenelson2lenelson2 Member
    523 karma

    Are you sure that's the right pt, section and problem #?

  • BinghamtonDaveBinghamtonDave Alum Member 🍌🍌
    8689 karma

    I just did 11-4-22 and it is not what you have referenced.

  • Sarah889Sarah889 Alum Member
    877 karma

    @lenelson2 said:
    Are you sure that's the right pt, section and problem #?

    Thanks for pointing that out! I just fixed it.

  • Sarah889Sarah889 Alum Member
    877 karma

    @BinghamtonDave said:
    I just did 11-4-22 and it is not what you have referenced.

    Sorry about that- I meant PT 12. I just fixed it.

  • apublicdisplayapublicdisplay Alum Member
    edited January 2017 696 karma

    This should be PT12.S4.Q22 not "PT11.S4.Q22."

    You understand how answer choice (C) resolves the paradox, but you're saying it's not a perfect explanation. But, I don't think it has to sufficiently (completely) resolve the paradox since the question stem says "which one of the following, if true, most helps to resolve the apparently conflicting findings."

  • lenelson2lenelson2 Member
    523 karma

    No worries about the pt # @bswise2 , I'm glad we figured it out!

    So I'm still working on resolve reconcile explain questions, so take this explanation with a grain of salt. But, this definitely is a tricky question. I can see how you wanted to focus on the weight gain, but to explain a paradox in a stimulus I think we can focus on either group in the comparison to explain the paradox, whether it be the group that maintained their weight or the group that gained weight in this case, even though the support given is only for the group that gained weight.

    I can also see why you aren't fond of the wording of C, but I don't think it's necessary to know how likely they are to gain or lose weight, we just need to know that they weigh less-hence, "tend to be leaner" than nonsmokers. The likelihood is irrelevant and I think if it stated "less likely to gain weight" we wouldn't be able to conclude that they actually weigh less than the nonsmokers. What do you think?

  • BinghamtonDaveBinghamtonDave Alum Member 🍌🍌
    8689 karma

    Yeah, every once in awhile there are questions where we simply have to choose the best of the bunch. At bottom, I don't really know why the LSAT tests this skill. The "Most helps" skill that is. For a further example of this, please refer to PT C2 Section 3 Question 15 (Deer). I always tell myself with these RRE questions, I'm not presenting a dissertation on this phenomena to a committee of scientists, I'm simple looking for something that most accounts for this. Sometimes, I have to work wrong to right.

    My digression aside, this is how I approached this question.
    I always plug RRE questions into a bare-bones form and get precisely clear on what it is I am doing.

    If______, then how is it that______?
    So with this question what we have at bottom is:

    If there are studies that say weight gain is bad, then how is it that this particular weight gain over time actually looks to be healthier than staying the same weight?

    Well first of all, there is a lot wrong with this comparison. Starting with the temporal element of the good weight gain ( half a kilogram per year) vs the nebulously defined statement about weight gain in the last sentence, simply called "weight gain." These could be considered the same thing, but they also could exist over a continuum. "Weight gain" seems to be the superset that contains the .5 KG, but it could also be drastic weight gains. As an aside, there was a boxer in the 1990s that could have been one of the greatest boxers of all time: Riddick Bowe, but in between fights, the man had drastic weight gains that adversely effected not only his health, but made re-conditioning for fights very very difficult. Our conclusion lumps all weight gain: including that of Bowe in which this very measured .5 kilogram per year gain with age.

    It is in that space where I thought a relative difference that can resolve this issue was going to reside. In my pre-phrase, I was aim for something that played on the difference between the super-set of weight gain broadly and this subset of measured gain with age. Maybe something like: with age a little bit of weight each year strengthens the individual's immune system? Maybe the excess weight in a very measured way each year aids in the prevention of flu?

    Instead they gave us (C). (C) seems to suggest that a resolution of this issue resides in the fact that smokers are messing up the numbers because they are lean... The inclusion of (C) seems to tell us that in the first study, the measured weight gain folks were more healthy than the static folks because the static folks were smokers... Which means that this study might lack the force that holds together this alleged discrepancy. Odd, but as an above commenter points out @apublicdisplay (C) does the most.

    I hope this helps. Any further RRE questions you come across that fit this mold, please post. They are a great curve breaker technique to be equipped with.

    David

Sign In or Register to comment.